Difference between revisions of "2003 AMC 12A Problems/Problem 24"

(Replaced content with ' == See Also == {{AMC12 box|year=2003|ab=A|num-b=23|after=25}}')
Line 1: Line 1:
 +
== Problem ==
 +
 +
If <math>a\geq b > 1,</math> what is the largest possible value of <math>\log_{a}(a/b) + \log_{b}(b/a)?</math>
 +
 +
<math>
 +
\mathrm{(A)}\ -2      \qquad
 +
\mathrm{(B)}\ 0    \qquad
 +
\mathrm{(C)}\ 2      \qquad
 +
\mathrm{(D)}\ 3      \qquad
 +
\mathrm{(E)}\ 4
 +
</math>
 +
 +
== Solution ==
 +
 +
Using logarithmic rules, we see that
 +
 +
<cmath>\log_{a}a-\log_{a}b+\log_{b}b-\log_{b}a = 2-(\log_{a}b+\log_{b}a)</cmath>
 +
<cmath>=2-(\log_{a}b+\frac {1}{\log_{a}b})</cmath>
 +
 +
Since <math>a</math> and <math>b</math> are both positive, using [[AM-GM]] gives that the term in parentheses must be at least <math>2</math>, so the largest possible values is <math>2-2=\boxed{0}.</math>
  
 
== See Also ==
 
== See Also ==
  
 
{{AMC12 box|year=2003|ab=A|num-b=23|after=25}}
 
{{AMC12 box|year=2003|ab=A|num-b=23|after=25}}

Revision as of 13:55, 28 February 2010

Problem

If $a\geq b > 1,$ what is the largest possible value of $\log_{a}(a/b) + \log_{b}(b/a)?$

$\mathrm{(A)}\ -2      \qquad \mathrm{(B)}\ 0     \qquad \mathrm{(C)}\ 2      \qquad \mathrm{(D)}\ 3      \qquad \mathrm{(E)}\ 4$

Solution

Using logarithmic rules, we see that

\[\log_{a}a-\log_{a}b+\log_{b}b-\log_{b}a = 2-(\log_{a}b+\log_{b}a)\] \[=2-(\log_{a}b+\frac {1}{\log_{a}b})\]

Since $a$ and $b$ are both positive, using AM-GM gives that the term in parentheses must be at least $2$, so the largest possible values is $2-2=\boxed{0}.$

See Also

2003 AMC 12A (ProblemsAnswer KeyResources)
Preceded by
Problem 23
Followed by
25
1 2 3 4 5 6 7 8 9 10 11 12 13 14 15 16 17 18 19 20 21 22 23 24 25
All AMC 12 Problems and Solutions